SBI-PO 2007 (REASONING)

Sunday 5 June, 2011


SBI-PO 2007 (REASONING)


Directions (1-5): In.the questions given below,, certain symbols are used with the following  meaning:
A @ B means A is greater than B.
A * B means A is either greater than or equal to B.
A # B means A is equal to B.
A $ B means A is either smaller than or equal to B.
A + B means A is smaller than B"
Now in each of the following questions assuming the given
statements to be true, find which of the . to conclusions I and II
given below them is/are definitely True?
Give answer (a) if only conclusion I is true
Give answer (b) if only conclusion II is true.
Give answer (c) if either conclusion I or II is true.
Give answer (d) if neither conclusion I nor II is true.
. Give answer (e) if both conclusions I and I I are true.
(1-5) @ ⇒ > * ⇒ ≥ # ⇒ =
# ⇒ ≤ + ⇒ <



l. Statements:
D + T ; E $ V ; F * E @ D
Conclusions
I. D $ V
II. D + F
Ans. (b) Statements
D + T, E $ V, F * T, E @ D
After conversion
D<t, E ≤ V, F ≥ T, E > D
or, V ≥ E > D < T ≤ F
Conclusions
I. D $ V ⇒ D ≤ V : Not true
D is smaller than V.
II. D + F ⇒ D < F : True

2. Statements :
B + D ; E $ T ; T * P ; P @ B
Conclusions :
I. P $ D
II. D @ F
Ans.(b) Statements
B + D, E $ T, T * P, P @ B
After conversion
B < D, E ≤ T, T P, P > B
or, E ≤ T ≥ P > B < D
Conclusion
I. P $ D ⇒ P ≤ D : Not True
II. P @ D ⇒ P > D : Not True
While considering the relation between two entities three possibilities exit : greater than, equal to or smaller than Therefore either I or
II is true.

3. Statements :
T * U ; U $ W; V @ L ; W + V
Conclusions :
I. V@T
II. L#W
Ans. (d) Statements
T * U, U $ W, V @ L, W + V
After conversion
T ≥ U,U ≤ W, V > L, W < V
or, T ≥ U, ≤ W < V > L
Conclusions
I. V @ T ⇒ V > T : Not True
II. L # W ⇒ L > W : Not True

4. Statements :
P # Q; N # M ; M @ R ; R * P
Conclusions :
I. P + N
II. Q Q $ M
Ans.(a) Statements
P $ Q, N # M, M @ R, R * P
After conversion
P ≤ Q, N M, M > R, R ≥ P
or, N = M > R ≥ P ≥ Q
Conclusions
I. P + N ⇒ P < N : True
II. Q $ W ⇒ Q ≤ M : Not True

5. Statements :
E * F ; G $ H ; H # E ; G @ K
Conclusion:
Ans.(e) Statements
E * F, G $ H, H # E, G @ K
After conversion
E ≥ F, G ≤ H H + E, G > K
or, K < G ≤ H = E ≥ F
I. H @ K ⇒ H > K : True
II. H * F ⇒ H ≥ K : True

Directions (6-ll): study the foil owing letter _number symbol sequence carefully and answer the questions given below:
3 D 5 F E 3 8 $ M 2 I K * P T @ U 9 A 7 1 £ H J 4 Q 6
(6-11) (1) There are altogether 27 elements in the above sequence.
(ii) There are only 14 letters in the above sequence.
(iii) There only 9 digits in the above sequence.
(iv) There are only 4 symbols in the above sequence.
(v) The middle terms of the sequence is P.

6. What should come in place of the question (?)in the following sequence?
5ES, MIP, ?, IHQ
(a) TUA
(b) TU7
(c) @ 9 1
(d) T91
(e) None of these
Ans.(b)  +6                +6             +6   
              5   →    M    →     T  →  1
       +6          +6         +6
     E   →  J     →   U    →    H
    +6        +6            +6
  $  →   P   →     7     →     Q
Therefore, ? = TU7

7. Which of the following is exactly in the midway between the eleventh from the left end and the 7th from the right end?
(a) P
(b) @
(c) T
(d) U
(e) None of these
Ans.(b) 11th from left and ⇒ I
7th from right and end ⇒ 1
Remaining  elements between I and 1:
K    *    P    T      @      U      9     A      7
                            ↓
           Middle Term

8. Which of the following is the sixth to the right of the twentieth from the right end?
(a) 5
(b) F
(c) P
(d) K
(e) None of these
Ans.(c) 6th to the right of the 20th from the right end means 20 - 6 = 14th from the right end.
There are altogether 27 element in the above sequence and the middle term, i.e., 14th element from either end is P. Therefore, our required answer answered answer is option (c)

9. How many such digits are there in the above sequence which are immediately preceded as well as followed by digits ?
(a) None
(b) One
(c) Two
(d) Three
(e) More than three
Ans.(a) Digit
Digit
Digit

10. If the first fifteen elements are written in the reverse order then which of the following will be eighth to the left of the thirteenth element from right end?
(a) M
(b) 8
(c) $
(d) *
(e) None of these.
Ans.(a) 8th to the left of 13th element from right end means 13 + 8 = 21st from the right end.
21 st element from the right end is equivalent to 28 - 21 = 7th element from the left and vice-versa.
1st ↔ 15th 2nd ↔ 14th
3rd ↔ 13th 4th ↔ 12 th
5th ↔ 11th 6th ↔ 10th
7th ↔ 9th
Therefore, required element would be 9th from the in original sequence.
9th from left → M.

11. If all consonants starling from B are given sequentially the value of even numbers such as B = 2, C = :4 and so on and all the vowels are given the value of 5 each, then what will be the value of the letters of the word CUSTOM?
(a) 92
(b) 86
(c) 82
(d) 96
(e) None of these
Ans.(d) According o question,
A

5
B

2
C

4
D

6
E

5
F

8
G

10
H

12
I

5
J

14
K

16
L

18
M

20

N

22
O

5
P

24
Q

26
R

28
S

30
T

32
U

5
V

34
W

36
X

38
Y

40
Z

42
Now,
C

4      +    
U

5     +    
S

30    +    
T

32    +     
O

5     +    20  
M
 ↓
20


=  96

12. How many such 5s are there in the following sequence that the sum of the two immediately following digits is greater than the sum of the two immediately preceding digits?
3 1 6 5 83 2 45 5 4 8 7 9 1 5 3 4 8 7 5 9 8 7 6 4
(a) One
(b) Two
(c) Three
(d) four
(e) None of these
Ans. (d)

13.If A + B means "A is the sister of B", A × B means "A is the wife of B", A ÷ B means "A is the father of B" and A - B means "A is the brother of B", then which of following expresses the relationship that "T is the daughter of P "?
(a) P × Q ÷ R +S - T
(b) P × Q ÷ R -T + S
(c) P × Q ÷ R + T - S
(d) P × Q ÷ R + S + T
(e) None of these
Ans.(b) Option (1)
P × Q ÷ R + S - T
P × Q → P is the wife od Q.
Q ÷ R → Q is the father of R.
R + S → R is the sister of S.
S - T → S is the brother of T.
Note : The sex of T is not known.
Deductions
(i) Q is the husband of P.
(ii) P is the mother of R,S and T.
(iii) Q is the father of R,S and T.
(iv) R is the sister of S and T.
(v) S. is the brother of R and T.
Option (2)
P × Q → P is the wife of Q.
Q ÷ R → Q is the father of T.
R - T → R is the brother of T.
T + S → T is the sister of S.
Note : The sex of S in not known.
Deductions
(i) Q is the husband of R
(ii) P is the mother of R, S and T.
(iii) Q is the father of R, S and T.
(iv) T is the daughter of P and Q.

14. If the position of the first letter of English alphabet is interchanged with the position of the four length letter. second letter with the fifteenth letter in such a way that M is interchanged with Z then which of the. following letters will be 9th to the right of 17th letter from,the right?
(a) F
(b)E
(c) R
(d) r
(e) None of these
Ans.(a) According to question, the new sequence would be :
N O P Q R S T U V W X Y Z A B C D E F G H I J K L M
9th to the right of 17th letter from the right means (17-9) = 8th letter from the right.
= 8th letter from right ⇒ F.

Directions (15-18): Read the 'following information and answer the questions given below:
(i) Seven friends P, Q, R, S, T, U and W have gathered at the Mumbai airport Five of them are scheduled to go five different places - Delhi' Chennai' Lucknow, Bangalore and Calcutta'
(ii) Five of them executives, each specialising in viz. Administration (Admin')'Human Resource management (HRM), Marketing' Systems .and finance.
(iii) T, an executive is going to Chennai and is neither from Finance nor Marketing.
(iv) W is system specialist and is leaving for Delhi' U is an executive but is not going to one-of the five places.
(v) Q is an executive but not HRM but has come at the airport to see his fiends.
(vi) P is un executive but not from Marketing and is flying to one of the destinations but not to Bangalore or Calcutta.
(15 - 18) : On the basis of given information and conclusions as well as sub-conclusions drawn from them we can construct the following chart:

Person
P
Q
R
S
T
U
W
Field
Finance
HRA
-
-
Administration
Marketing
System
Destination
Lucknow
-
Culcutta/Bangalore
Culcutta/Bangalore
Chennai
-
Delhi

15. Who is going to fly to Bangalore?.
(a) Data inadequate
(b) R
(c) S
(d) P
(e) None of these
Ans. (a)

16. Who among the following specializes in Marketing?
(a) S
(b)P
(c) U
(d) Data inadequate
(e) None of these
Ans.(c)

17. R has specializes in which of the following fields?
(a) Finance
(b) Marketing
(c) Either Marketing or Finance
(d) None
(e) All of these
Ans. (d)

18. The one who is going to fly to Chennai is
(a) Not an executive
(b) From administer
(c) S
(d) From finance
(e) None of these
Ans. (b)

19. How many pairs of letters are there in the word 'NURSING' which have as letters between them as in the alphabet ?
(a) one
(b) Three
(c) Five
(d) Six
(e) None of these
Ans.(b)

There are three such pairs .

Directions (20-26) Read the following information and answer the questions given below'
A famous museum issues entry passes to all its visitors for security reasons' visitors" are allowed in batches after every one hour. In a day there are six batches' A code is printed on entry pass which keeps on changing for every.atch' Following is an illustration of pass-code issued for each batch.
Batch I:
clothes neat and clean liked are all by
Batch II:
by clothes neat all are and clean liked
Batch III:
liked by clothes clean and neat all are and so on.
(20-26) : On careful analysis of the given pass-codes it is evident that the, last word of the previous code becomes the first word for the next batch and the first and second words shifted to the second and the third positions respectively. Again the seventh and sixthwords occupy the fourth and fifth positions respectively and then the third, the fourth and the fifth words of the previous code are written in the same order in the pass-code for the next batch.
Thus,
Pass code for Batch I
1
clothes
2
neat
3
and
4
clean
5
liked
6
are
7
all
8
by
Pass code for Batch II
8
by
1
1
clothes
2
2
neat
3
7
all
4
6
are
5
3
and
6
4
clean
7
5
liked
8
Pass code for Batch III
8
liked
1
1
by
2
2
clothes
3
7
clean
4
6
and
5
3
neat
6
4
all
7
5
are
8
Pass code for Batch VI
8
are
1
1
liked
2
2
by
3
7
all
4
6
neat
5
3
clothes
6
4
clean
7
5
and
8
Pass code for Batch V
8
and
1
1
and
2
2
liked
3
7
clean
4
6
clothes
5
3
by
6
4
all
7
5
neat
8
Pass code for Batch VI
8
neat
1
and
2
are
7
all
6
by
3
liked
4
clean
5
clothes
Now on the basis of above analysis we can summa rise the procedure for getting Pass code for each subsequent batch with reference to the Pass code for Batch I in the following manner:
Pass code for Batch I
1
clothes
2
neat
3
and
4
clean
5
liked
6
are
7
all
8
by
Pass code for Batch II
8
by
1
clothes
2
neat
7
all
6
are
3
and
4
clean
5
liked
Pass code for Batch III
5
liked
8
by
1
clothes
4
clean
3
and
2
neat
7
all
6
are
Pass code for Batch VI
6
are
5
liked
8
by
7
all
2
neat
1
clothes
4
clean
3
and
Pass code for Batch V
3
and
6
are
5
liked
4
clean
1
clothes
8
by
7
all
2
neat
Pass code for Batch VI
2
neat
3
and
6
are
7
all
8
by
5
liked
4
clean
1
clothes
Thus, we can write any required step i.e. Pass code for any batch from the given pass-code directly.

20. If pass-code for the third batch is 'night succeed day and hard work to for', what will be the pass-code for the sixth batch ?
(a) work hard to for succeed night and day
(b) hard work for and succeed night to day
(c) work hard for to succeed night and day
(d) hard work for to succeed night and day
(e) None of these
Ans. (c) Pass code for Batch III
5
night
8
succeed
1
day
4
and
3
hard
2
work
7
to
6
for
Pass code for Batch VI
2
work
3
hard
6
for
7
to
8
succeed
5
night
4
and
1
day

21. If 'visit in zoo should the we time day' is the the fifth batch, 'zoo we the should visit day time in 'will be the pass-code for which of the following batches ?
(a) Il
(b) IV
(c) I
(d) III
(e) VI
Ans.(d) Pass code for Batch V
3
visit
6
in
5
zoo
4
should
1
the
8
we
7
time
2
day
Given Pass code
5
zoo
8
we
1
the
4
should
3
and
2
day
7
time
6
in
Clearly, this is the poss code for Batch III.

22. Sanjay visited the museum in the fourth batch and was issued a pass-code 'to fast rush avoid not do very run. What would have been the pass-code for him had he visited he museum in the second batch?
(a) rush do not avoid to run very fast
(b) rush not do avoid to run very fast
(c) avoid rush not do to run very fast
(d) Data inadequate
(e) None of these
Ans.(a) Pass code for Batch IV
6
to
5
fast
8
rush
7
avoid
2
not
1
do
4
very
3
run
Pass code for Batch II
8
rush
1
do
2
not
7
avoid
6
to
3
run
4
very
5
fast

23. Subodh went to visit the museum in the second batch. He was issued a pass-code 'length the day equal of and and night are' However, he could not visit the museum in the second batch as he was little late. He then preferred to visit in the fifth batch. What will be the new pass-code issued to him?
(a) and of are night the length equal day
(b) and are of night the length equal day
(c) and of are night the equal day length
(d) and of are the night length day equal
(e) None of these
Ans.(a) Pass code for Batch II
8
length
1
the
2
day
7
equal
6
of
3
and
4
night
5
are
Pass code for Batch IV
3
and
6
of
5
are
4
night
1
the
8
length
7
equal
2
day

24. If pass-code for the second batch is 'to confidence hard you leads work and success', what will be the pass-code for the fourth batch ?
(a) leads success to you hard confidence and work
(b) leads success you to hard confidence and work
(c) leads success to you hard confidence work and
(d) leads to success you hard confidence work and
(e) None of these
Ans. (a) Pass code for Batch II
8
to
1
confidence
2
hard
7
you
6
leads
3
work
4
and
5
success
Pass code for Batch IV
6
leads
5
success
8
to
7
you
2
hard
1
confidence
4
and
3
work

25, lf the pass-code issued for the last (sixth) batch is 'and pencil by all boys used are pen. What will be the pass-code for the of,first batch?
(a) pencil and pen are used by all boys
(b) pen and pencil used are by all boys
(c) pen and pencil are used by all boys
(d) pencil and pen are used all by boys
(e) None of these
Ans.(c) Pass code for Batch VI
2
and
3
pencil
6
by
7
all
8
boys
5
used
4
are
1
pen
Pass code for Batch I
1
pen
2
and
3
pencil
4
are
5
used
6
by
7
all
8
boys

26. If the pass-code for the sixth batch is 'not go the way to of out do', what will be the pass-code for the third batch?
(a) of do to out go not way the
(b) of to do out not go way the
(c) of to go out do not way the
(d)Data inadequate
(e) None of these
Ans.(e) Pass code for Batch IV
6
not
5
go
8
the
7
way
2
to
1
of
4
out
3
do
Pass code for Batch III
5
of
8
to
1
do
4
out
3
go
2
not
7
way
6
the

Directions (27-32): In each question below are given three statements. followed b1' four conclusions-I, II, III and IV' You have to take the given statements to be true even if they seem to be at variance with commonly known .facts.Read all the conclusions and then decide which of the given conclusions logically follow (s) from the given statements disregarding commonly known facts'

27. Statements
Some books are Pens.
All Pens are chairs.
Some chairs are tables'
Conclusions
I. Some books are chairs'
II. Some chairs are books'
III. All tables are chairs'
IV. Some tables are chairs'
(a) All follow
(b) Only I, II, and III follow
(c) Only I, II and IV follow
(d) Only II, III and IV follow
(e) None of these
Ans. (c) First and third premises are Particular Affirmative, i.e. I-type.
Second premise is Universal Affirmative (A-type).
1. Some books are pens.

2. All pens are chairs.
we know that, I + A ⇒ I - type conclusion
Therefore, out derive conclusion would be:
"Some books are chairs"
This is the conclusion I.
Conclusion II is the conversion of out derived conclusion.
Two out of three premises are Particular and hence, Universal conclusion is invalid. That is, conclusion III does not follow.
Conclusion IV is the conversion of the third premise.
Therefore, only conversions I, II and IV follow.

28. Statements
All cars are jeeps
All jeePs are buses.
All buses are trucks.
Conclusions
I. All trucks are buses.
IL AII buses are jeeps.
III. All jeeps are cars.
IV. All cars are trucks.
(a) None follows
(b) All follow
(c) OnlY III and lV follow
(d) Only IV follows
(e) None of these
Ans.(d) All the three premises are Universal Affirmative (A-type).
All cars are jeeps.

All jeeps are buses.
We know,
A + A ⇒ A- type conclusion
Therefore, our required conclusion would be :
"All cars are buses."
There is no such conclusion.
Now,
All cars are buses.

All buses are trucks.
All buses are trucks.
We know that,
A + A ⇒ A - type conclusion
Thus, our derived conclusion would be :
"All cars are trucks "
This is the conclusion IV.
Again,
All jeeps are buses.

All buses are trucks.
We know that,
A + A ⇒ A - type conclusion
Thus, our derived conclusion would be
"All jeeps are truks"
Therefore, only conclusion IV follows.
Thus, our required answer is option (d).


29, Statements
Some trees are flowers'
Some flowers are Pencils'
Some Pencils are tables'
Conclusions
I. Some tables are flowers'
II. Some Pencils are trees'
III. Some tables are trees'
IV. Some trees are Pencils'
(a) All follow
(b) None follows
(c) Only I and III follows
(d) Only Il and IV follow
(e) None of these
Ans.(b) All the three premises are Particular Affirmative (I-type). Therefore, no conclusion can be derived from these premises.
Now look for any conversion and/or implication : There is no such conclusion.

30. Statements
All rods are bricks.
Some bricks are ropes.
All ropes are doors'
Conclusions
I. Some rods are doors.
II. Some doors are bricks.
III. Some rods are not doors'
IV. All doors are ropes.
(a) Only I and II follow
(b) Only I, II and III follow
(c) Only either I or III and II follow
(d) Only either I or III and IV follow
(e) None of these
Ans.(e) First premise is Universal Affirmative (A-type). Second premise is Particular Affirmative (1-type). Third premise is Universal
Affirmative (A-type).
Some bricks are ropes.

All ropes are doors.
We know that,
I + A ⇒ 1-type conclusion
Conclusion : Some bricks are doors, It is conversion of conclusion II.
Conclusions I and III from complementary pair. Therefore either conclusion I or III follows.
Therefore, our required answer is option (c)

31. Statements
Some books are Pens.
Some pens are watches.
All watches are radios.
Conclusions
I. Some radios are watches'
II. Some radios are Pens'
III. Some watches are books'
IV. Some books are watches'
(a) All follow (b) Only I and III follow
(c) only Il and IV follow (d) Only I and IV follow
(e) None of these
Ans. (e) First and second premises are Particular Affirmative i.e, 1-type. Third premise is Universal Affirmative (A-type).
Conclusion I is the conversion of the third premise. Second and third premises are relevant for the conclusion II.
Thus,
Some pens are watches.

All watches are radios.
We know that,
I + A ⇒ 1-type conclusion
Conclusion : Some pens are radios, conclusion II is the conversion of this conclusion. Therefore, only conclusion I and II follow.


32. Statements
All towns are villages.
No village is forest.
Some forests are rivers'
Conclusions
I. Some forests are villages'
Il. Some forests are not villages'
ll. Some rivers are not villages'
IV. All villages are towns'
(a) All follow
(b) OnlY either I or 11 follows
(c) OnlY either I or Il and III follow
(d) None follows
(e) None of these
Ans. (e) First premise → Universal Affirmative (A-type).
Second premise → Universal Negative (E-type).
Third premise → Particular Affirmative (1-type).
Now,
All towns are villages.

No village is forest.
We know that,
A + E ⇒ E-type conclusion
Conclusion :
No town is forest.
There is no such conclusion,
All towns are villages.

No village is forest.
We know that, E + I ⇒ O 1 - type conclusion
Thus, our derived conclusion would be :
"Some rivers are not village."
This is the conclusion III.
Again,
No town is forest.

Some forest are rivers.
We know that,
E + I ⇒ O1 - type conclusion
Conclusion:
Some rivers are not towns.
There is no such conclusion.
Conclusion II is the conversion of the second premise.
Therefore, Conclusions II and III follow.

33. ln a row of boys facing north, Sudhanshu is twelfth from his left. When shifted to his right by four places' the becomes eighteenth from the right end of the row' How many boys are there in the row?
(a) 32
(b) 33
(c) 34
(d) Data inadequate
(e) None of these
Ans.(b)

After shirting his place

Total number of boys in the row = (16 + 18) - 1 = 33

34. In a certain code language PROBLEM is written as MPERLOB. How will NUMBERS be written in that code?
(a) SNUREMB
(b) SNRUBME
(c) SNRUEMB
(d) SNRUMEB
(e) None of these
Ans.(c)
1
P
2
R
3
O
4
B
5
L
6
E
7
M
It has been coded as
7
M
1
P
6
E
2
R
5
L
3
O
4
B
Similarly,
1
N
2
U
3
M
4
B
5
E
6
R
7
S
Its code would be :
7
N
1
R
6
U
2
E
5
M
3
B
4 S

Directions (35-40): In each question below is given a statement .followed by two assumptions numbered I and II. An assumption is something supposed or taken for granted. You have to consider the statement and the following assumptions and decide which of the assumptions is implicit in the statement.
Give answer (a) if only assumption I is implicit.
Give answer (b) if only assumption lI,is implicit.
Give answer (c) if either I or II is implicit.
Give answer (d) if neither I nor ll is implicit.
Give answer (e) if both I and II are implicit.

35. Statement: The Government has recently hiked the prices of diesel and petrol to reduce the oil pool deficit.Assumptions:
l. The amount earned by this increase may be substantial enough to reduce the deficit.
II. There may be wide spread protests against the price hike.
Ans.(e) Both the assumptions are implicit in the statement. It is clearly mentioned in the statement that the Government has liked theprices of diesel and petrol to reduce the oil pool deficit. Whenever the prices of commodities are hiked, generally people raised the voiceagainst such measure.

36. Statement: The X passenger car manufacturing company announced a sharp reduction in the prices of their luxury cars.
Assumptions:
I. There may be an increase in the sale of their luxury cars.
II. The other.such car manufacturers may also reduce their prices.
Ans.(e) The price of any product is lowered assuming that its-demand will increase. Therefore, assumption I is implicit in the
statement,

37. Statement: A foreign film producer rendered his apology before Indian society for misinterpreting a par-t of Indian epic.
Assumptions:
I. Indians are very sensitive to the misinterpretation of their epic.
II. It is possible to derive wrong meaning I1om the epic.
Ans.(e) From the content of the statement it is is clear that the both the assumptions are implicit in the statement.

38. Statement: Aswin's mother instructed him to return home by train if it rains heavily.
Assumptions
I. Aswin may not be to decide himself if it rains heavily.
II. The trains may ply even if it rains heavily.
Ans.(b) Only assumption II is implicit in the statement. If Aswin's mother asked his son to return home by train if it rains heavily, it
implies that the trains would ply even if it rains heavily.

39. Statement: The Government of India has decided to start atrack II dialogue with its neighbour to reduce tension in the area.
Assumptions:
I. The neighbouring country may agree to participate in the track II dialogue.
II. The people involved in track II dialogue may be able to persuade their respective Governments.
Ans.(e) Both the assumptions are implicit in the statement.

40. Statement: The host in one of the popular T.V. programmes announced that the channel will contact the viewers between 9.00 a.m. to 6.00 p.m on weekdays and the lucky ones will be given fabulous prizes.
Assumptions:
I. The people may remain indoors to receive the phone call.
II. More people may start watching the programme.
Ans.(e) Both the assumption' are implicit in the statement.

Directions (41-45): In making decision about important questions, it is desirable to be able to distinguish between "Strong"
arguments and ,,Welt', arguments so.fqr as the lt relate to the question."Strong,, arguments are those which are both important and directly related to the question. ,,We&k,, arguments are those which are of minor importance and also may not be directly related to the collusion or may be related to a trivial aspect of the question Instructions: Each question below is followed by a statement and two arguments numbered I and II. you have to decide which of the arguments is a ,,Strong,' argument and which is a "Wreath" argument.
Give answer (a) if only argument I is strong.
Give answer (b) if only argument II is strong.
Give answer (c) if either I or II is strong.
Give answer (d) if neither I nor II is strong.
Give answer (e) if both I and II are strong.

41. Statement: Should the habit of late coming in educational institutions be checked?
Arguments:
I. No. Until it affects the work.
II. Yes. Discipline must be maintained.
Ans.(b) Only argument II is strong.

42. Statement: Should seniority be the only criterion for the promotion?
Arguments:
I. No. All the senior employees are not interested in promotion.
II. Yes. Otherwise senior employees do feel humiliated.
Ans.(d) Neither of the argument is strong.

43, Statement: Should children be prevented completely from watching television?
Arguments:
I. No. We get vital information regarding education through television.
II. Yes. It hampers the study of children.
Ans.(a) Only argument I is strong. Now-a-days television is an essential means to provide useful academic information. Therefore, it is not desirable to prevent children from watching detected programmes on television. For the same reason, argument II is invalid.

44. Statement: Should trade unions be banned completely?
Arguments:
I. No. This is the only way through which employees can put their demands before management.
II. Yes. Employees get their illegal demands fulfilled through these unions.
Ans.(a) Only argument I is strong. It is true that employee put their genuine demands before the management through the trade unions. Therefore, it is not judicious to ban the trade unions completely. It is true that employees compel the management in some instances to concede their some illegal demands through the unions but the solution suggested is not appropriate.

45. Statement: Should women be given equal opportunity in matter of employment in every field?
Arguments:
I. Yes. They are equally capable.
II. No. They have to shoulder household responsibilities too.
Ans.(a) Only argument I is strong. Women are equally capable and they should be given equal opportunity in matter of employment. Argument II makes no point.

Directions (46-50): In each question below is given a statement followed by two conclusions numbered I and II. You have to assume everything in the statement to be true, then consider the two conclusions together and decide which of them logically follows beyond a reasonable doubt from the information given in the
statement.
Give answer (a) if only conclusion I follows.
Give answer (b) if only conclusion II follows.
Give answer (c) if either I or II follows.
Give answer (d) if neither I nor II follows, and
Give answer (e) if both I and II follow

46. Statement: The cabinet of state X took certain steps to tackle the milk glut in.the state as the cooperatives and Government dairies failed to use the available milk - A new report
Conclusions :
I . The milk production of State .X, is more than its need.
II. The Government and co-operative dairies in State ,X, are not equipped in terms of resources and technology to such and technology to handle such excess milk.
Ans. (e) Both the conclusions logically follows from the information given in the statement. The term milk glut clearly indicates that the milk production of State 'X' is more than its requirement. Again, it is clearly mentioned that the Government and co-operative dairies in State 'X' failed to use the available milk - it implies that such dairies in State 'X' are not equipped properly.

47. Statement: It has been decided or the Government to withdraw 33% of the subsidy on cooking gas from the beginning of next month - A spokesman of the Government.
Conclusions:
I. People now no more desire of need such subsidy from Government as they can afford increased price of the cooking gas.
II. The price of the cooking gas will increase at least by 33% from the next month.
Ans. (d) Neither conclusion I nor conclusion II follows. The subsidy on any item cannot be 100 per cent. It is mentioned in the statement the Government the Government has decided to withdraw 33 per cent of "the subsidy" on cooking gas and not 33 per cent of the actual price of the cooking gas. Therefore, it is erroneous to assume that the price of the cooking gas will increase at least by 33 per cent . Any policy of the Government affects all sections of the society and it cannot be assumed that poor people do not need subsidy. Thus, neither of the conclusions follows.

48. Statement: .,The Government will review the present policy of the diesel price in view of further in the international oil prices" - A spokesman of the Government.
Conclusions:
I. The Government will increase the price of the diesel after the imminent spurt in the international oil prices.
II. The Government will not increase the price of the diesel even after the imminent spilt in the international oil prices.
Ans.(c) From the statement it is clear that the Government will either increase the price o0f diesel or will not increase the price of diesel. The deficit on this count can be adjusted by some other means.Therefore, after the review of the present policy of the diesel price in view of further spurt in the international oil prices, the Government is left with only two options - to increases or not to increase the price of diesel. Therefore, either conclusion I or conclusion II follows.

49. Statement: My first and foremost task is to beautify this city - if City 'X' and Y can do it - why can't we do it - Statement of Municipal Commissioner of City 'Z' after taking over charge.
Conclusions :
I. The people of City ,Z, are not aware about the present state of ugliness of their city.
II. The present Commissioner has worked in City ,X, and Y and has good experience of beautifying cities.
Ans. (d) Neither of the conclusions follows. It clear from the statement that the city 'Z' requires beautification but this does not employ that the people of that city are unaware about the state of ugliness of their city. Therefore, conclusion I does not follow. The new Municipal Commissioners that the task of beautification of city 'Z' could also be accomplished if such has happened in City 'X' and City 'Y'. from this it cannot be derived that Municipal Commissioner has served in city 'X' and 'Y'. While asserting something one may quote the work of another person. Hence, conclusion II is also not valid.

50. Statement: Women,s Organisation, it India have welcomed the amendment of the Industrial Employment Rules 1946 to curb sexual harassment at the work place.
Conclusions:
I. Sexual harassment of women at work place is more prevalent in lndia as compared to other developed countries. II. Many organisation's in India will stop recruiting women to avoid such problems.
Ans.(d) Neither conclusion I nor conclusion II follows. In the absence of harsh punitive measured the cases of sexual harassment may take place any where. Therefore, it is erroneous to assume that sexual harassment of women at work place is more prevalent in India as compared to other developed countries. The statement exhorts that amendment in the Act is meant to curb sexual harassment at the work place and it cannot be taken as the imposition of restriction on recruitment of women. In order to avoid pain in arms we cannot our arms.

Directions (51-55): Each of the questions below consists of a question and two statements numbered I and II given below it. you have to decide whether the.data provided in the statement are sufficient .to answer the question. Read both the statements and -
Give answer (a) if the data in, statement I alone are sufficient to answer the question,
Give answer (b) if the data in statement Il alone are sufficient to answer the question, while, the data in statement I alone are not sufficient to answer the question.
Give answer (c) if the data either in the statement I alone or in statement II alone are sufficient to answer the question.
Give answer (d) if the data even in both statements I and II together are not sufficient to answer the question.
Give answer (e) if the data in both statements I and II together are necessary to answer the question.

51. In which direction is Ravi facing?
I. Ashok is to the right of Ravi.
II. Samir is sifting opposite of Ashok facing north.
Ans. (d) It is not given the statement I that Ashok or Ravi is facing towards which direction. Therefore, it is not possible to determine the direction of right side of Ravi.
From statements II
Samir is sitting opposite of Ashok facing north. It implies that Ashok is facing toward south.
From both the Statements
Ashok is facing toward south and is sitting to the right of Ravi.
It implies that Ravi is facing toward south or north.
Therefore, are required answer is (d)

52. How M is related to K ?
I. The sister of K is the mother of N who is daughter of M.
II. P is the sister of M.
Ans.(e) From statement I
The mother of N is the daughter of M.
The mother of N is the sister of K.
Therefore, the mother of N and K are the children of M.
But it is not clear where M is the father or mother of K.
From statement II
P is the wife of M. Therefore, M is the husband of P.
There is no information about K in statement II.
From both the statements it is dear that M is the father of K

53. Is D brother of T ?
I. T is the sister of M and K.
II. K is the brother of D.
Ans. (d) From statement I
T is the sister of M and K.
There is no information about D in the statement I.
It is not clear whether M and K are males or females.
Thus, the data inn statement I alone are not sufficient to answer the question.
From statement II
K is the brother of D.
There is no information about T in the statement U.
We can't determine the sex of D even with the help of data given in the statements I and II.
Therefore, our required answer is option (d)

54. How many sons does P have ?
I. S and T are brothers of M.
II. The mother of T is P who has only one daughter.
Ans. (e) There is no information about P in the statement.
Therefore, the data in statement I alone are not sufficient to answer the question.
But it is clear that M, S and T are brother and/or sisters to one another.
From statement II
P is the mother of T and she has only one daughter.
It implies that T is the daughter of P. From statement 1 and II P has two son S and T. M is the daughter of P. Therefore, the data given in both the statements I and II are necessary to answer the question.

55. Who is the tallest among P,Q, R, S and T?
I. R is taller than Q and T.
II. T is taller than S and P and S is taller than Q and R.
Ans. (b) From statement I R > Q, T
There is no information about P and S in the statement I.
Thus, the data given in the statement I alone are not sufficient to answer the question.
From statement II
T > S, P and
S > Q, R
or T > S > Q, R
         
It is clear that T is the tallest among them.
Thus, the data in the statement II alone are sufficient to answer the question, while the data in the statement I atone are not sufficient
to answer the question.

Directions (56-60): Below is given a passage followed by several possible inferences which can be drawn.from the facts stated in the passage. You have to examine each inference separately in the context of the passage and decide upon its degree of truth or falsity.
Mark answer (a) if the inference is definitely true" i.e., it properly follows from the statement of facts given.
Mark answer (b) if the inference is "probably true,,though not "definitely true" in the light of the facts given.
Mark answer (c) if the data are inadequate i.e. from the facts given you cannot say whether the inference is likely to be true or false.
Mark answer (d) if the inference is "probably false" though not "definitely false" in the light of the facts given.
Mark answer (e) if the inference is ,,definitely false" i.e., it cannot possibly be drawn from the facts given or it contradicts the given facts.
With the purpose of upliftment of Gonda district in Uttar Pradesh, a new formula was developed for practical success in several fields, such as, irrigation, animal husbandry dairy farming, moral uplift and creation of financial resources. Small farms were clustered for irrigation by one diesel pump which could irrigate about 20 acres of land. Youth were prompted to take loans from the banks for purchase of engine pumps to be supplied to the farmers on rent. This formula worked so well that the villages in Gonda district were saturated with irrigation facilities. cattle
rearing was linked with multiple cropping
. Most of the targets fixed for different areas were achieved, which was an unusual phenomenon. This could be possible only because of right motivation, participation and initiative of the people. imagination and creativity
combined together helped in finding out workable solutions to the problems of the community'


56. There was no problem and complaint of people residing in entire Gonda district, before the beginning of the project'
Ans.(e)

57. Purchasing of engine pumps by individual farmers may be beyond their affordable limits.
Ans.(a)

58. Earlier farming was not basically one of the professions of people in Gonda district.
Ans.(e)

59. By using the same formula upliftment of any other district is possible.
Ans.(a)

60. There are very few people who can motivate others in the right direction.
Ans.(c)

Directions (61-65): In each of these questions there are two sets of figures. The figures on thc top are Problem Figures (four figures and one question-marked space) and those on the bottom are Answer Figures indicated by numbers a, b, c' d and e' A series is established if one of the five Answer Figures is placed at the "question-marked space". Figures form a series it they change from left to right according to some rule' The
number of the Answer figure which should be placed in question-marked space is the answer. All the five figures i.e., four Problem Figures and one Answer Figure placed in the question-marked space should be considered as forming the series.


61. Problem Figures

Ans.(d) The movement and charges of designs from Problem Figure (a) to (b) can be shown as :

From Problem Figure (d) to (e)

From Problem Figure (b) to (c) similar charges would occur as that have been occurred from Problem Figure (d) to (e) and a new design(P) would appear at the lower middle position because starting from Problem Figure (b) a new design appears in the clockwise direction in the subsequent figures.

62. Problem Figures

Ans. (a) From Problem Figure (d) to (e) the lower design is reversed laterally while the other design moves to the opposite side. Similar changes would occur from Problem Figure (b) to (c).

63. Problem Figures

Ans. (b) In each , subsequent figure all the designs moves one step in clockwise direction and one of the designs gets paired.

64. Problem Figures

Ans.(c) In each subsequent figure the place of designs rotates through 45° anticlockwise. From Problem Figure (d) to (e) the fourthdesign from right is shifted to the first position while all other designs move one step and the last design is replaced by a new design. Similar changes would occur from Problem Figure (b) to (c).

65. Problem Figures

Ans.(a) Each of the arcs rotates 90° clockwise or anticlockwise and 180° alternately.

Directions (66-70) In each of the following questions a series beings with figure on the extreme left. One of the five numbered figures in the series does not belong to the series. The two numbered figures one each on extreme left and the extern right fit into the series. You have to taken as many aspect into accounts as possible of the figures in the series and find out the one and only on of the five numbered figures which does not fit into the series. The number of that figure is the answer.

66.

Ans. (d) The movement of designs from unnumbered figure to figure (a) can be shown as :

The movement of designs from figure (a) to figure (b) can be shown as :

These two steps are repeated alternately. In figure (d) three should be (P) instead of (S).

67.

Ans. (e) The last figure inverted from of the first figure. Figure (d) is the inverted from of figure (b). Therefore, figure (e) should be inverted from of figure (a).

68.

Ans.(c) In the first step one line segment is deleted from the upper figure while the lower figure is inverted. In the next step one line segment is deleted from the lower figure while the upper figure is inverted. These two steps are repeated alternately. In the figure (c) both the figures are inverted.

69.

Ans.(a) The triangle is rotating respectively 45° , 90°, 135°, 180°, 225° ............ in anticlockwise direction while the arc is inverted in the next figure. In figure (a) triangle has been rotated through 90° anticlockwise instead of 90° clockwise.

70.

Ans.(d) The line segments are added in the following manner :
 0 → 1 1 → 2     2 → 3    3  →   4   4  →   5    5  →  6
Upper 1 1 0 1 1 1
Side
Lower 1 2 1 1 2 0
Side
Therefore, figure (d) is the wrong in the series.

Directions (71 - 75) : In each of the question given below which one of the five answer figures on the right should come after the problem figures on the left, if the sequence were continued ?

71. Problem Figures

Answer Figure

Ans.(b) In the subsequent figure respectively one, two three ............ design (s) is/are inverted.

72. Problem Figures

Answer Figure

Ans. (c) In the subsequent figures respectively two, three, two, four ................. line segments are deleted.

73. Problem Figures

Answer Figure

Ans.(a) In each subsequent figure all the designs move half step in anticlockwise direction, two designs interchange positions and oneof these two designs with a new design.

74. Problem Figures

Answer Figure

Ans. (c) The complete design is inverted in the next figure and one design is added in every figure.

75. Problem Figures

Answer Figure

Ans. (e) From Problem Figure (a) to (b) the following changes occur.

Similar changes would occur from Problem Figure (5) to Answer Figure.



SBI-PO 2007 (REASONING)


Directions (1-5): In.the questions given below,, certain symbols are used with the following  meaning:
A @ B means A is greater than B.
A * B means A is either greater than or equal to B.
A # B means A is equal to B.
A $ B means A is either smaller than or equal to B.
A + B means A is smaller than B"
Now in each of the following questions assuming the given
statements to be true, find which of the . to conclusions I and II
given below them is/are definitely True?
Give answer (a) if only conclusion I is true
Give answer (b) if only conclusion II is true.
Give answer (c) if either conclusion I or II is true.
Give answer (d) if neither conclusion I nor II is true.
. Give answer (e) if both conclusions I and I I are true.
(1-5) @ ⇒ > * ⇒ ≥ # ⇒ =
# ⇒ ≤ + ⇒ <



l. Statements:
D + T ; E $ V ; F * E @ D
Conclusions
I. D $ V
II. D + F
Ans. (b) Statements
D + T, E $ V, F * T, E @ D
After conversion
D<t, E ≤ V, F ≥ T, E > D
or, V ≥ E > D < T ≤ F
Conclusions
I. D $ V ⇒ D ≤ V : Not true
D is smaller than V.
II. D + F ⇒ D < F : True

2. Statements :
B + D ; E $ T ; T * P ; P @ B
Conclusions :
I. P $ D
II. D @ F
Ans.(b) Statements
B + D, E $ T, T * P, P @ B
After conversion
B < D, E ≤ T, T P, P > B
or, E ≤ T ≥ P > B < D
Conclusion
I. P $ D ⇒ P ≤ D : Not True
II. P @ D ⇒ P > D : Not True
While considering the relation between two entities three possibilities exit : greater than, equal to or smaller than Therefore either I or
II is true.

3. Statements :
T * U ; U $ W; V @ L ; W + V
Conclusions :
I. V@T
II. L#W
Ans. (d) Statements
T * U, U $ W, V @ L, W + V
After conversion
T ≥ U,U ≤ W, V > L, W < V
or, T ≥ U, ≤ W < V > L
Conclusions
I. V @ T ⇒ V > T : Not True
II. L # W ⇒ L > W : Not True

4. Statements :
P # Q; N # M ; M @ R ; R * P
Conclusions :
I. P + N
II. Q Q $ M
Ans.(a) Statements
P $ Q, N # M, M @ R, R * P
After conversion
P ≤ Q, N M, M > R, R ≥ P
or, N = M > R ≥ P ≥ Q
Conclusions
I. P + N ⇒ P < N : True
II. Q $ W ⇒ Q ≤ M : Not True

5. Statements :
E * F ; G $ H ; H # E ; G @ K
Conclusion:
Ans.(e) Statements
E * F, G $ H, H # E, G @ K
After conversion
E ≥ F, G ≤ H H + E, G > K
or, K < G ≤ H = E ≥ F
I. H @ K ⇒ H > K : True
II. H * F ⇒ H ≥ K : True

Directions (6-ll): study the foil owing letter _number symbol sequence carefully and answer the questions given below:
3 D 5 F E 3 8 $ M 2 I K * P T @ U 9 A 7 1 £ H J 4 Q 6
(6-11) (1) There are altogether 27 elements in the above sequence.
(ii) There are only 14 letters in the above sequence.
(iii) There only 9 digits in the above sequence.
(iv) There are only 4 symbols in the above sequence.
(v) The middle terms of the sequence is P.

6. What should come in place of the question (?)in the following sequence?
5ES, MIP, ?, IHQ
(a) TUA
(b) TU7
(c) @ 9 1
(d) T91
(e) None of these
Ans.(b)  +6                +6             +6   
              5   →    M    →     T  →  1
       +6          +6         +6
     E   →  J     →   U    →    H
    +6        +6            +6
  $  →   P   →     7     →     Q
Therefore, ? = TU7

7. Which of the following is exactly in the midway between the eleventh from the left end and the 7th from the right end?
(a) P
(b) @
(c) T
(d) U
(e) None of these
Ans.(b) 11th from left and ⇒ I
7th from right and end ⇒ 1
Remaining  elements between I and 1:
K    *    P    T      @      U      9     A      7
                            ↓
           Middle Term

8. Which of the following is the sixth to the right of the twentieth from the right end?
(a) 5
(b) F
(c) P
(d) K
(e) None of these
Ans.(c) 6th to the right of the 20th from the right end means 20 - 6 = 14th from the right end.
There are altogether 27 element in the above sequence and the middle term, i.e., 14th element from either end is P. Therefore, our required answer answered answer is option (c)

9. How many such digits are there in the above sequence which are immediately preceded as well as followed by digits ?
(a) None
(b) One
(c) Two
(d) Three
(e) More than three
Ans.(a) Digit
Digit
Digit

10. If the first fifteen elements are written in the reverse order then which of the following will be eighth to the left of the thirteenth element from right end?
(a) M
(b) 8
(c) $
(d) *
(e) None of these.
Ans.(a) 8th to the left of 13th element from right end means 13 + 8 = 21st from the right end.
21 st element from the right end is equivalent to 28 - 21 = 7th element from the left and vice-versa.
1st ↔ 15th 2nd ↔ 14th
3rd ↔ 13th 4th ↔ 12 th
5th ↔ 11th 6th ↔ 10th
7th ↔ 9th
Therefore, required element would be 9th from the in original sequence.
9th from left → M.

11. If all consonants starling from B are given sequentially the value of even numbers such as B = 2, C = :4 and so on and all the vowels are given the value of 5 each, then what will be the value of the letters of the word CUSTOM?
(a) 92
(b) 86
(c) 82
(d) 96
(e) None of these
Ans.(d) According o question,
A

5
B

2
C

4
D

6
E

5
F

8
G

10
H

12
I

5
J

14
K

16
L

18
M

20

N

22
O

5
P

24
Q

26
R

28
S

30
T

32
U

5
V

34
W

36
X

38
Y

40
Z

42
Now,
C

4      +    
U

5     +    
S

30    +    
T

32    +     
O

5     +    20  
M
 ↓
20


=  96

12. How many such 5s are there in the following sequence that the sum of the two immediately following digits is greater than the sum of the two immediately preceding digits?
3 1 6 5 83 2 45 5 4 8 7 9 1 5 3 4 8 7 5 9 8 7 6 4
(a) One
(b) Two
(c) Three
(d) four
(e) None of these
Ans. (d)

13.If A + B means "A is the sister of B", A × B means "A is the wife of B", A ÷ B means "A is the father of B" and A - B means "A is the brother of B", then which of following expresses the relationship that "T is the daughter of P "?
(a) P × Q ÷ R +S - T
(b) P × Q ÷ R -T + S
(c) P × Q ÷ R + T - S
(d) P × Q ÷ R + S + T
(e) None of these
Ans.(b) Option (1)
P × Q ÷ R + S - T
P × Q → P is the wife od Q.
Q ÷ R → Q is the father of R.
R + S → R is the sister of S.
S - T → S is the brother of T.
Note : The sex of T is not known.
Deductions
(i) Q is the husband of P.
(ii) P is the mother of R,S and T.
(iii) Q is the father of R,S and T.
(iv) R is the sister of S and T.
(v) S. is the brother of R and T.
Option (2)
P × Q → P is the wife of Q.
Q ÷ R → Q is the father of T.
R - T → R is the brother of T.
T + S → T is the sister of S.
Note : The sex of S in not known.
Deductions
(i) Q is the husband of R
(ii) P is the mother of R, S and T.
(iii) Q is the father of R, S and T.
(iv) T is the daughter of P and Q.

14. If the position of the first letter of English alphabet is interchanged with the position of the four length letter. second letter with the fifteenth letter in such a way that M is interchanged with Z then which of the. following letters will be 9th to the right of 17th letter from,the right?
(a) F
(b)E
(c) R
(d) r
(e) None of these
Ans.(a) According to question, the new sequence would be :
N O P Q R S T U V W X Y Z A B C D E F G H I J K L M
9th to the right of 17th letter from the right means (17-9) = 8th letter from the right.
= 8th letter from right ⇒ F.

Directions (15-18): Read the 'following information and answer the questions given below:
(i) Seven friends P, Q, R, S, T, U and W have gathered at the Mumbai airport Five of them are scheduled to go five different places - Delhi' Chennai' Lucknow, Bangalore and Calcutta'
(ii) Five of them executives, each specialising in viz. Administration (Admin')'Human Resource management (HRM), Marketing' Systems .and finance.
(iii) T, an executive is going to Chennai and is neither from Finance nor Marketing.
(iv) W is system specialist and is leaving for Delhi' U is an executive but is not going to one-of the five places.
(v) Q is an executive but not HRM but has come at the airport to see his fiends.
(vi) P is un executive but not from Marketing and is flying to one of the destinations but not to Bangalore or Calcutta.
(15 - 18) : On the basis of given information and conclusions as well as sub-conclusions drawn from them we can construct the following chart:

Person
P
Q
R
S
T
U
W
Field
Finance
HRA
-
-
Administration
Marketing
System
Destination
Lucknow
-
Culcutta/Bangalore
Culcutta/Bangalore
Chennai
-
Delhi

15. Who is going to fly to Bangalore?.
(a) Data inadequate
(b) R
(c) S
(d) P
(e) None of these
Ans. (a)

16. Who among the following specializes in Marketing?
(a) S
(b)P
(c) U
(d) Data inadequate
(e) None of these
Ans.(c)

17. R has specializes in which of the following fields?
(a) Finance
(b) Marketing
(c) Either Marketing or Finance
(d) None
(e) All of these
Ans. (d)

18. The one who is going to fly to Chennai is
(a) Not an executive
(b) From administer
(c) S
(d) From finance
(e) None of these
Ans. (b)

19. How many pairs of letters are there in the word 'NURSING' which have as letters between them as in the alphabet ?
(a) one
(b) Three
(c) Five
(d) Six
(e) None of these
Ans.(b)

There are three such pairs .

Directions (20-26) Read the following information and answer the questions given below'
A famous museum issues entry passes to all its visitors for security reasons' visitors" are allowed in batches after every one hour. In a day there are six batches' A code is printed on entry pass which keeps on changing for every.atch' Following is an illustration of pass-code issued for each batch.
Batch I:
clothes neat and clean liked are all by
Batch II:
by clothes neat all are and clean liked
Batch III:
liked by clothes clean and neat all are and so on.
(20-26) : On careful analysis of the given pass-codes it is evident that the, last word of the previous code becomes the first word for the next batch and the first and second words shifted to the second and the third positions respectively. Again the seventh and sixthwords occupy the fourth and fifth positions respectively and then the third, the fourth and the fifth words of the previous code are written in the same order in the pass-code for the next batch.
Thus,
Pass code for Batch I
1
clothes
2
neat
3
and
4
clean
5
liked
6
are
7
all
8
by
Pass code for Batch II
8
by
1
1
clothes
2
2
neat
3
7
all
4
6
are
5
3
and
6
4
clean
7
5
liked
8
Pass code for Batch III
8
liked
1
1
by
2
2
clothes
3
7
clean
4
6
and
5
3
neat
6
4
all
7
5
are
8
Pass code for Batch VI
8
are
1
1
liked
2
2
by
3
7
all
4
6
neat
5
3
clothes
6
4
clean
7
5
and
8
Pass code for Batch V
8
and
1
1
and
2
2
liked
3
7
clean
4
6
clothes
5
3
by
6
4
all
7
5
neat
8
Pass code for Batch VI
8
neat
1
and
2
are
7
all
6
by
3
liked
4
clean
5
clothes
Now on the basis of above analysis we can summa rise the procedure for getting Pass code for each subsequent batch with reference to the Pass code for Batch I in the following manner:
Pass code for Batch I
1
clothes
2
neat
3
and
4
clean
5
liked
6
are
7
all
8
by
Pass code for Batch II
8
by
1
clothes
2
neat
7
all
6
are
3
and
4
clean
5
liked
Pass code for Batch III
5
liked
8
by
1
clothes
4
clean
3
and
2
neat
7
all
6
are
Pass code for Batch VI
6
are
5
liked
8
by
7
all
2
neat
1
clothes
4
clean
3
and
Pass code for Batch V
3
and
6
are
5
liked
4
clean
1
clothes
8
by
7
all
2
neat
Pass code for Batch VI
2
neat
3
and
6
are
7
all
8
by
5
liked
4
clean
1
clothes
Thus, we can write any required step i.e. Pass code for any batch from the given pass-code directly.

20. If pass-code for the third batch is 'night succeed day and hard work to for', what will be the pass-code for the sixth batch ?
(a) work hard to for succeed night and day
(b) hard work for and succeed night to day
(c) work hard for to succeed night and day
(d) hard work for to succeed night and day
(e) None of these
Ans. (c) Pass code for Batch III
5
night
8
succeed
1
day
4
and
3
hard
2
work
7
to
6
for
Pass code for Batch VI
2
work
3
hard
6
for
7
to
8
succeed
5
night
4
and
1
day

21. If 'visit in zoo should the we time day' is the the fifth batch, 'zoo we the should visit day time in 'will be the pass-code for which of the following batches ?
(a) Il
(b) IV
(c) I
(d) III
(e) VI
Ans.(d) Pass code for Batch V
3
visit
6
in
5
zoo
4
should
1
the
8
we
7
time
2
day
Given Pass code
5
zoo
8
we
1
the
4
should
3
and
2
day
7
time
6
in
Clearly, this is the poss code for Batch III.

22. Sanjay visited the museum in the fourth batch and was issued a pass-code 'to fast rush avoid not do very run. What would have been the pass-code for him had he visited he museum in the second batch?
(a) rush do not avoid to run very fast
(b) rush not do avoid to run very fast
(c) avoid rush not do to run very fast
(d) Data inadequate
(e) None of these
Ans.(a) Pass code for Batch IV
6
to
5
fast
8
rush
7
avoid
2
not
1
do
4
very
3
run
Pass code for Batch II
8
rush
1
do
2
not
7
avoid
6
to
3
run
4
very
5
fast

23. Subodh went to visit the museum in the second batch. He was issued a pass-code 'length the day equal of and and night are' However, he could not visit the museum in the second batch as he was little late. He then preferred to visit in the fifth batch. What will be the new pass-code issued to him?
(a) and of are night the length equal day
(b) and are of night the length equal day
(c) and of are night the equal day length
(d) and of are the night length day equal
(e) None of these
Ans.(a) Pass code for Batch II
8
length
1
the
2
day
7
equal
6
of
3
and
4
night
5
are
Pass code for Batch IV
3
and
6
of
5
are
4
night
1
the
8
length
7
equal
2
day

24. If pass-code for the second batch is 'to confidence hard you leads work and success', what will be the pass-code for the fourth batch ?
(a) leads success to you hard confidence and work
(b) leads success you to hard confidence and work
(c) leads success to you hard confidence work and
(d) leads to success you hard confidence work and
(e) None of these
Ans. (a) Pass code for Batch II
8
to
1
confidence
2
hard
7
you
6
leads
3
work
4
and
5
success
Pass code for Batch IV
6
leads
5
success
8
to
7
you
2
hard
1
confidence
4
and
3
work

25, lf the pass-code issued for the last (sixth) batch is 'and pencil by all boys used are pen. What will be the pass-code for the of,first batch?
(a) pencil and pen are used by all boys
(b) pen and pencil used are by all boys
(c) pen and pencil are used by all boys
(d) pencil and pen are used all by boys
(e) None of these
Ans.(c) Pass code for Batch VI
2
and
3
pencil
6
by
7
all
8
boys
5
used
4
are
1
pen
Pass code for Batch I
1
pen
2
and
3
pencil
4
are
5
used
6
by
7
all
8
boys

26. If the pass-code for the sixth batch is 'not go the way to of out do', what will be the pass-code for the third batch?
(a) of do to out go not way the
(b) of to do out not go way the
(c) of to go out do not way the
(d)Data inadequate
(e) None of these
Ans.(e) Pass code for Batch IV
6
not
5
go
8
the
7
way
2
to
1
of
4
out
3
do
Pass code for Batch III
5
of
8
to
1
do
4
out
3
go
2
not
7
way
6
the

Directions (27-32): In each question below are given three statements. followed b1' four conclusions-I, II, III and IV' You have to take the given statements to be true even if they seem to be at variance with commonly known .facts.Read all the conclusions and then decide which of the given conclusions logically follow (s) from the given statements disregarding commonly known facts'

27. Statements
Some books are Pens.
All Pens are chairs.
Some chairs are tables'
Conclusions
I. Some books are chairs'
II. Some chairs are books'
III. All tables are chairs'
IV. Some tables are chairs'
(a) All follow
(b) Only I, II, and III follow
(c) Only I, II and IV follow
(d) Only II, III and IV follow
(e) None of these
Ans. (c) First and third premises are Particular Affirmative, i.e. I-type.
Second premise is Universal Affirmative (A-type).
1. Some books are pens.

2. All pens are chairs.
we know that, I + A ⇒ I - type conclusion
Therefore, out derive conclusion would be:
"Some books are chairs"
This is the conclusion I.
Conclusion II is the conversion of out derived conclusion.
Two out of three premises are Particular and hence, Universal conclusion is invalid. That is, conclusion III does not follow.
Conclusion IV is the conversion of the third premise.
Therefore, only conversions I, II and IV follow.

28. Statements
All cars are jeeps
All jeePs are buses.
All buses are trucks.
Conclusions
I. All trucks are buses.
IL AII buses are jeeps.
III. All jeeps are cars.
IV. All cars are trucks.
(a) None follows
(b) All follow
(c) OnlY III and lV follow
(d) Only IV follows
(e) None of these
Ans.(d) All the three premises are Universal Affirmative (A-type).
All cars are jeeps.

All jeeps are buses.
We know,
A + A ⇒ A- type conclusion
Therefore, our required conclusion would be :
"All cars are buses."
There is no such conclusion.
Now,
All cars are buses.

All buses are trucks.
All buses are trucks.
We know that,
A + A ⇒ A - type conclusion
Thus, our derived conclusion would be :
"All cars are trucks "
This is the conclusion IV.
Again,
All jeeps are buses.

All buses are trucks.
We know that,
A + A ⇒ A - type conclusion
Thus, our derived conclusion would be
"All jeeps are truks"
Therefore, only conclusion IV follows.
Thus, our required answer is option (d).


29, Statements
Some trees are flowers'
Some flowers are Pencils'
Some Pencils are tables'
Conclusions
I. Some tables are flowers'
II. Some Pencils are trees'
III. Some tables are trees'
IV. Some trees are Pencils'
(a) All follow
(b) None follows
(c) Only I and III follows
(d) Only Il and IV follow
(e) None of these
Ans.(b) All the three premises are Particular Affirmative (I-type). Therefore, no conclusion can be derived from these premises.
Now look for any conversion and/or implication : There is no such conclusion.

30. Statements
All rods are bricks.
Some bricks are ropes.
All ropes are doors'
Conclusions
I. Some rods are doors.
II. Some doors are bricks.
III. Some rods are not doors'
IV. All doors are ropes.
(a) Only I and II follow
(b) Only I, II and III follow
(c) Only either I or III and II follow
(d) Only either I or III and IV follow
(e) None of these
Ans.(e) First premise is Universal Affirmative (A-type). Second premise is Particular Affirmative (1-type). Third premise is Universal
Affirmative (A-type).
Some bricks are ropes.

All ropes are doors.
We know that,
I + A ⇒ 1-type conclusion
Conclusion : Some bricks are doors, It is conversion of conclusion II.
Conclusions I and III from complementary pair. Therefore either conclusion I or III follows.
Therefore, our required answer is option (c)

31. Statements
Some books are Pens.
Some pens are watches.
All watches are radios.
Conclusions
I. Some radios are watches'
II. Some radios are Pens'
III. Some watches are books'
IV. Some books are watches'
(a) All follow (b) Only I and III follow
(c) only Il and IV follow (d) Only I and IV follow
(e) None of these
Ans. (e) First and second premises are Particular Affirmative i.e, 1-type. Third premise is Universal Affirmative (A-type).
Conclusion I is the conversion of the third premise. Second and third premises are relevant for the conclusion II.
Thus,
Some pens are watches.

All watches are radios.
We know that,
I + A ⇒ 1-type conclusion
Conclusion : Some pens are radios, conclusion II is the conversion of this conclusion. Therefore, only conclusion I and II follow.


32. Statements
All towns are villages.
No village is forest.
Some forests are rivers'
Conclusions
I. Some forests are villages'
Il. Some forests are not villages'
ll. Some rivers are not villages'
IV. All villages are towns'
(a) All follow
(b) OnlY either I or 11 follows
(c) OnlY either I or Il and III follow
(d) None follows
(e) None of these
Ans. (e) First premise → Universal Affirmative (A-type).
Second premise → Universal Negative (E-type).
Third premise → Particular Affirmative (1-type).
Now,
All towns are villages.

No village is forest.
We know that,
A + E ⇒ E-type conclusion
Conclusion :
No town is forest.
There is no such conclusion,
All towns are villages.

No village is forest.
We know that, E + I ⇒ O 1 - type conclusion
Thus, our derived conclusion would be :
"Some rivers are not village."
This is the conclusion III.
Again,
No town is forest.

Some forest are rivers.
We know that,
E + I ⇒ O1 - type conclusion
Conclusion:
Some rivers are not towns.
There is no such conclusion.
Conclusion II is the conversion of the second premise.
Therefore, Conclusions II and III follow.

33. ln a row of boys facing north, Sudhanshu is twelfth from his left. When shifted to his right by four places' the becomes eighteenth from the right end of the row' How many boys are there in the row?
(a) 32
(b) 33
(c) 34
(d) Data inadequate
(e) None of these
Ans.(b)

After shirting his place

Total number of boys in the row = (16 + 18) - 1 = 33

34. In a certain code language PROBLEM is written as MPERLOB. How will NUMBERS be written in that code?
(a) SNUREMB
(b) SNRUBME
(c) SNRUEMB
(d) SNRUMEB
(e) None of these
Ans.(c)
1
P
2
R
3
O
4
B
5
L
6
E
7
M
It has been coded as
7
M
1
P
6
E
2
R
5
L
3
O
4
B
Similarly,
1
N
2
U
3
M
4
B
5
E
6
R
7
S
Its code would be :
7
N
1
R
6
U
2
E
5
M
3
B
4 S

Directions (35-40): In each question below is given a statement .followed by two assumptions numbered I and II. An assumption is something supposed or taken for granted. You have to consider the statement and the following assumptions and decide which of the assumptions is implicit in the statement.
Give answer (a) if only assumption I is implicit.
Give answer (b) if only assumption lI,is implicit.
Give answer (c) if either I or II is implicit.
Give answer (d) if neither I nor ll is implicit.
Give answer (e) if both I and II are implicit.

35. Statement: The Government has recently hiked the prices of diesel and petrol to reduce the oil pool deficit.Assumptions:
l. The amount earned by this increase may be substantial enough to reduce the deficit.
II. There may be wide spread protests against the price hike.
Ans.(e) Both the assumptions are implicit in the statement. It is clearly mentioned in the statement that the Government has liked theprices of diesel and petrol to reduce the oil pool deficit. Whenever the prices of commodities are hiked, generally people raised the voiceagainst such measure.

36. Statement: The X passenger car manufacturing company announced a sharp reduction in the prices of their luxury cars.
Assumptions:
I. There may be an increase in the sale of their luxury cars.
II. The other.such car manufacturers may also reduce their prices.
Ans.(e) The price of any product is lowered assuming that its-demand will increase. Therefore, assumption I is implicit in the
statement,

37. Statement: A foreign film producer rendered his apology before Indian society for misinterpreting a par-t of Indian epic.
Assumptions:
I. Indians are very sensitive to the misinterpretation of their epic.
II. It is possible to derive wrong meaning I1om the epic.
Ans.(e) From the content of the statement it is is clear that the both the assumptions are implicit in the statement.

38. Statement: Aswin's mother instructed him to return home by train if it rains heavily.
Assumptions
I. Aswin may not be to decide himself if it rains heavily.
II. The trains may ply even if it rains heavily.
Ans.(b) Only assumption II is implicit in the statement. If Aswin's mother asked his son to return home by train if it rains heavily, it
implies that the trains would ply even if it rains heavily.

39. Statement: The Government of India has decided to start atrack II dialogue with its neighbour to reduce tension in the area.
Assumptions:
I. The neighbouring country may agree to participate in the track II dialogue.
II. The people involved in track II dialogue may be able to persuade their respective Governments.
Ans.(e) Both the assumptions are implicit in the statement.

40. Statement: The host in one of the popular T.V. programmes announced that the channel will contact the viewers between 9.00 a.m. to 6.00 p.m on weekdays and the lucky ones will be given fabulous prizes.
Assumptions:
I. The people may remain indoors to receive the phone call.
II. More people may start watching the programme.
Ans.(e) Both the assumption' are implicit in the statement.

Directions (41-45): In making decision about important questions, it is desirable to be able to distinguish between "Strong"
arguments and ,,Welt', arguments so.fqr as the lt relate to the question."Strong,, arguments are those which are both important and directly related to the question. ,,We&k,, arguments are those which are of minor importance and also may not be directly related to the collusion or may be related to a trivial aspect of the question Instructions: Each question below is followed by a statement and two arguments numbered I and II. you have to decide which of the arguments is a ,,Strong,' argument and which is a "Wreath" argument.
Give answer (a) if only argument I is strong.
Give answer (b) if only argument II is strong.
Give answer (c) if either I or II is strong.
Give answer (d) if neither I nor II is strong.
Give answer (e) if both I and II are strong.

41. Statement: Should the habit of late coming in educational institutions be checked?
Arguments:
I. No. Until it affects the work.
II. Yes. Discipline must be maintained.
Ans.(b) Only argument II is strong.

42. Statement: Should seniority be the only criterion for the promotion?
Arguments:
I. No. All the senior employees are not interested in promotion.
II. Yes. Otherwise senior employees do feel humiliated.
Ans.(d) Neither of the argument is strong.

43, Statement: Should children be prevented completely from watching television?
Arguments:
I. No. We get vital information regarding education through television.
II. Yes. It hampers the study of children.
Ans.(a) Only argument I is strong. Now-a-days television is an essential means to provide useful academic information. Therefore, it is not desirable to prevent children from watching detected programmes on television. For the same reason, argument II is invalid.

44. Statement: Should trade unions be banned completely?
Arguments:
I. No. This is the only way through which employees can put their demands before management.
II. Yes. Employees get their illegal demands fulfilled through these unions.
Ans.(a) Only argument I is strong. It is true that employee put their genuine demands before the management through the trade unions. Therefore, it is not judicious to ban the trade unions completely. It is true that employees compel the management in some instances to concede their some illegal demands through the unions but the solution suggested is not appropriate.

45. Statement: Should women be given equal opportunity in matter of employment in every field?
Arguments:
I. Yes. They are equally capable.
II. No. They have to shoulder household responsibilities too.
Ans.(a) Only argument I is strong. Women are equally capable and they should be given equal opportunity in matter of employment. Argument II makes no point.

Directions (46-50): In each question below is given a statement followed by two conclusions numbered I and II. You have to assume everything in the statement to be true, then consider the two conclusions together and decide which of them logically follows beyond a reasonable doubt from the information given in the
statement.
Give answer (a) if only conclusion I follows.
Give answer (b) if only conclusion II follows.
Give answer (c) if either I or II follows.
Give answer (d) if neither I nor II follows, and
Give answer (e) if both I and II follow

46. Statement: The cabinet of state X took certain steps to tackle the milk glut in.the state as the cooperatives and Government dairies failed to use the available milk - A new report
Conclusions :
I . The milk production of State .X, is more than its need.
II. The Government and co-operative dairies in State ,X, are not equipped in terms of resources and technology to such and technology to handle such excess milk.
Ans. (e) Both the conclusions logically follows from the information given in the statement. The term milk glut clearly indicates that the milk production of State 'X' is more than its requirement. Again, it is clearly mentioned that the Government and co-operative dairies in State 'X' failed to use the available milk - it implies that such dairies in State 'X' are not equipped properly.

47. Statement: It has been decided or the Government to withdraw 33% of the subsidy on cooking gas from the beginning of next month - A spokesman of the Government.
Conclusions:
I. People now no more desire of need such subsidy from Government as they can afford increased price of the cooking gas.
II. The price of the cooking gas will increase at least by 33% from the next month.
Ans. (d) Neither conclusion I nor conclusion II follows. The subsidy on any item cannot be 100 per cent. It is mentioned in the statement the Government the Government has decided to withdraw 33 per cent of "the subsidy" on cooking gas and not 33 per cent of the actual price of the cooking gas. Therefore, it is erroneous to assume that the price of the cooking gas will increase at least by 33 per cent . Any policy of the Government affects all sections of the society and it cannot be assumed that poor people do not need subsidy. Thus, neither of the conclusions follows.

48. Statement: .,The Government will review the present policy of the diesel price in view of further in the international oil prices" - A spokesman of the Government.
Conclusions:
I. The Government will increase the price of the diesel after the imminent spurt in the international oil prices.
II. The Government will not increase the price of the diesel even after the imminent spilt in the international oil prices.
Ans.(c) From the statement it is clear that the Government will either increase the price o0f diesel or will not increase the price of diesel. The deficit on this count can be adjusted by some other means.Therefore, after the review of the present policy of the diesel price in view of further spurt in the international oil prices, the Government is left with only two options - to increases or not to increase the price of diesel. Therefore, either conclusion I or conclusion II follows.

49. Statement: My first and foremost task is to beautify this city - if City 'X' and Y can do it - why can't we do it - Statement of Municipal Commissioner of City 'Z' after taking over charge.
Conclusions :
I. The people of City ,Z, are not aware about the present state of ugliness of their city.
II. The present Commissioner has worked in City ,X, and Y and has good experience of beautifying cities.
Ans. (d) Neither of the conclusions follows. It clear from the statement that the city 'Z' requires beautification but this does not employ that the people of that city are unaware about the state of ugliness of their city. Therefore, conclusion I does not follow. The new Municipal Commissioners that the task of beautification of city 'Z' could also be accomplished if such has happened in City 'X' and City 'Y'. from this it cannot be derived that Municipal Commissioner has served in city 'X' and 'Y'. While asserting something one may quote the work of another person. Hence, conclusion II is also not valid.

50. Statement: Women,s Organisation, it India have welcomed the amendment of the Industrial Employment Rules 1946 to curb sexual harassment at the work place.
Conclusions:
I. Sexual harassment of women at work place is more prevalent in lndia as compared to other developed countries. II. Many organisation's in India will stop recruiting women to avoid such problems.
Ans.(d) Neither conclusion I nor conclusion II follows. In the absence of harsh punitive measured the cases of sexual harassment may take place any where. Therefore, it is erroneous to assume that sexual harassment of women at work place is more prevalent in India as compared to other developed countries. The statement exhorts that amendment in the Act is meant to curb sexual harassment at the work place and it cannot be taken as the imposition of restriction on recruitment of women. In order to avoid pain in arms we cannot our arms.

Directions (51-55): Each of the questions below consists of a question and two statements numbered I and II given below it. you have to decide whether the.data provided in the statement are sufficient .to answer the question. Read both the statements and -
Give answer (a) if the data in, statement I alone are sufficient to answer the question,
Give answer (b) if the data in statement Il alone are sufficient to answer the question, while, the data in statement I alone are not sufficient to answer the question.
Give answer (c) if the data either in the statement I alone or in statement II alone are sufficient to answer the question.
Give answer (d) if the data even in both statements I and II together are not sufficient to answer the question.
Give answer (e) if the data in both statements I and II together are necessary to answer the question.

51. In which direction is Ravi facing?
I. Ashok is to the right of Ravi.
II. Samir is sifting opposite of Ashok facing north.
Ans. (d) It is not given the statement I that Ashok or Ravi is facing towards which direction. Therefore, it is not possible to determine the direction of right side of Ravi.
From statements II
Samir is sitting opposite of Ashok facing north. It implies that Ashok is facing toward south.
From both the Statements
Ashok is facing toward south and is sitting to the right of Ravi.
It implies that Ravi is facing toward south or north.
Therefore, are required answer is (d)

52. How M is related to K ?
I. The sister of K is the mother of N who is daughter of M.
II. P is the sister of M.
Ans.(e) From statement I
The mother of N is the daughter of M.
The mother of N is the sister of K.
Therefore, the mother of N and K are the children of M.
But it is not clear where M is the father or mother of K.
From statement II
P is the wife of M. Therefore, M is the husband of P.
There is no information about K in statement II.
From both the statements it is dear that M is the father of K

53. Is D brother of T ?
I. T is the sister of M and K.
II. K is the brother of D.
Ans. (d) From statement I
T is the sister of M and K.
There is no information about D in the statement I.
It is not clear whether M and K are males or females.
Thus, the data inn statement I alone are not sufficient to answer the question.
From statement II
K is the brother of D.
There is no information about T in the statement U.
We can't determine the sex of D even with the help of data given in the statements I and II.
Therefore, our required answer is option (d)

54. How many sons does P have ?
I. S and T are brothers of M.
II. The mother of T is P who has only one daughter.
Ans. (e) There is no information about P in the statement.
Therefore, the data in statement I alone are not sufficient to answer the question.
But it is clear that M, S and T are brother and/or sisters to one another.
From statement II
P is the mother of T and she has only one daughter.
It implies that T is the daughter of P. From statement 1 and II P has two son S and T. M is the daughter of P. Therefore, the data given in both the statements I and II are necessary to answer the question.

55. Who is the tallest among P,Q, R, S and T?
I. R is taller than Q and T.
II. T is taller than S and P and S is taller than Q and R.
Ans. (b) From statement I R > Q, T
There is no information about P and S in the statement I.
Thus, the data given in the statement I alone are not sufficient to answer the question.
From statement II
T > S, P and
S > Q, R
or T > S > Q, R
         
It is clear that T is the tallest among them.
Thus, the data in the statement II alone are sufficient to answer the question, while the data in the statement I atone are not sufficient
to answer the question.

Directions (56-60): Below is given a passage followed by several possible inferences which can be drawn.from the facts stated in the passage. You have to examine each inference separately in the context of the passage and decide upon its degree of truth or falsity.
Mark answer (a) if the inference is definitely true" i.e., it properly follows from the statement of facts given.
Mark answer (b) if the inference is "probably true,,though not "definitely true" in the light of the facts given.
Mark answer (c) if the data are inadequate i.e. from the facts given you cannot say whether the inference is likely to be true or false.
Mark answer (d) if the inference is "probably false" though not "definitely false" in the light of the facts given.
Mark answer (e) if the inference is ,,definitely false" i.e., it cannot possibly be drawn from the facts given or it contradicts the given facts.
With the purpose of upliftment of Gonda district in Uttar Pradesh, a new formula was developed for practical success in several fields, such as, irrigation, animal husbandry dairy farming, moral uplift and creation of financial resources. Small farms were clustered for irrigation by one diesel pump which could irrigate about 20 acres of land. Youth were prompted to take loans from the banks for purchase of engine pumps to be supplied to the farmers on rent. This formula worked so well that the villages in Gonda district were saturated with irrigation facilities. cattle
rearing was linked with multiple cropping
. Most of the targets fixed for different areas were achieved, which was an unusual phenomenon. This could be possible only because of right motivation, participation and initiative of the people. imagination and creativity
combined together helped in finding out workable solutions to the problems of the community'


56. There was no problem and complaint of people residing in entire Gonda district, before the beginning of the project'
Ans.(e)

57. Purchasing of engine pumps by individual farmers may be beyond their affordable limits.
Ans.(a)

58. Earlier farming was not basically one of the professions of people in Gonda district.
Ans.(e)

59. By using the same formula upliftment of any other district is possible.
Ans.(a)

60. There are very few people who can motivate others in the right direction.
Ans.(c)

Directions (61-65): In each of these questions there are two sets of figures. The figures on thc top are Problem Figures (four figures and one question-marked space) and those on the bottom are Answer Figures indicated by numbers a, b, c' d and e' A series is established if one of the five Answer Figures is placed at the "question-marked space". Figures form a series it they change from left to right according to some rule' The
number of the Answer figure which should be placed in question-marked space is the answer. All the five figures i.e., four Problem Figures and one Answer Figure placed in the question-marked space should be considered as forming the series.


61. Problem Figures

Ans.(d) The movement and charges of designs from Problem Figure (a) to (b) can be shown as :

From Problem Figure (d) to (e)

From Problem Figure (b) to (c) similar charges would occur as that have been occurred from Problem Figure (d) to (e) and a new design(P) would appear at the lower middle position because starting from Problem Figure (b) a new design appears in the clockwise direction in the subsequent figures.

62. Problem Figures

Ans. (a) From Problem Figure (d) to (e) the lower design is reversed laterally while the other design moves to the opposite side. Similar changes would occur from Problem Figure (b) to (c).

63. Problem Figures

Ans. (b) In each , subsequent figure all the designs moves one step in clockwise direction and one of the designs gets paired.

64. Problem Figures

Ans.(c) In each subsequent figure the place of designs rotates through 45° anticlockwise. From Problem Figure (d) to (e) the fourthdesign from right is shifted to the first position while all other designs move one step and the last design is replaced by a new design. Similar changes would occur from Problem Figure (b) to (c).

65. Problem Figures

Ans.(a) Each of the arcs rotates 90° clockwise or anticlockwise and 180° alternately.

Directions (66-70) In each of the following questions a series beings with figure on the extreme left. One of the five numbered figures in the series does not belong to the series. The two numbered figures one each on extreme left and the extern right fit into the series. You have to taken as many aspect into accounts as possible of the figures in the series and find out the one and only on of the five numbered figures which does not fit into the series. The number of that figure is the answer.

66.

Ans. (d) The movement of designs from unnumbered figure to figure (a) can be shown as :

The movement of designs from figure (a) to figure (b) can be shown as :

These two steps are repeated alternately. In figure (d) three should be (P) instead of (S).

67.

Ans. (e) The last figure inverted from of the first figure. Figure (d) is the inverted from of figure (b). Therefore, figure (e) should be inverted from of figure (a).

68.

Ans.(c) In the first step one line segment is deleted from the upper figure while the lower figure is inverted. In the next step one line segment is deleted from the lower figure while the upper figure is inverted. These two steps are repeated alternately. In the figure (c) both the figures are inverted.

69.

Ans.(a) The triangle is rotating respectively 45° , 90°, 135°, 180°, 225° ............ in anticlockwise direction while the arc is inverted in the next figure. In figure (a) triangle has been rotated through 90° anticlockwise instead of 90° clockwise.

70.

Ans.(d) The line segments are added in the following manner :
 0 → 1 1 → 2     2 → 3    3  →   4   4  →   5    5  →  6
Upper 1 1 0 1 1 1
Side
Lower 1 2 1 1 2 0
Side
Therefore, figure (d) is the wrong in the series.

Directions (71 - 75) : In each of the question given below which one of the five answer figures on the right should come after the problem figures on the left, if the sequence were continued ?

71. Problem Figures

Answer Figure

Ans.(b) In the subsequent figure respectively one, two three ............ design (s) is/are inverted.

72. Problem Figures

Answer Figure

Ans. (c) In the subsequent figures respectively two, three, two, four ................. line segments are deleted.

73. Problem Figures

Answer Figure

Ans.(a) In each subsequent figure all the designs move half step in anticlockwise direction, two designs interchange positions and oneof these two designs with a new design.

74. Problem Figures

Answer Figure

Ans. (c) The complete design is inverted in the next figure and one design is added in every figure.

75. Problem Figures

Answer Figure

Ans. (e) From Problem Figure (a) to (b) the following changes occur.

Similar changes would occur from Problem Figure (5) to Answer Figure.


0 comments:

Post a Comment

 
 
 

Followers

 
Blogger Widgets